Web Paint-by-Number Forum
Comments on Puzzle #22428: The answer
By Thomas Genuine (Genuine)

peek at solution       solve puzzle
  quality:   difficulty:   solvability: line & color logic only  

Puzzle Description Suppressed:Click below to view spoilers

#1: Jacqueline Vigliano (jiggyhh) on Jul 24, 2013 [SPOILER]

Comment Suppressed:Click below to view spoilers
#2: Thomas Genuine (Genuine) on Jul 24, 2013
OK Jacqueline, thx... :)

That angle is arctg2 - arctg0.5 = 36,87°
#3: marjorie rex (mamo) on Jul 25, 2013
I'll take your word for it.
#4: amanda_jamma (amanda_jamma) on Aug 3, 2013 [SPOILER]
Comment Suppressed:Click below to view spoilers

Show: Spoilers

Goto next topic

You must register and log in to be able to participate in this discussion.